Infinite sum of harmonic numberSum of reciprocals of numbers with certain terms omittedWhat is the closed...

Is it possible to make sharp wind that can cut stuff from afar?

Motorized valve interfering with button?

When blogging recipes, how can I support both readers who want the narrative/journey and ones who want the printer-friendly recipe?

What Brexit solution does the DUP want?

How to make payment on the internet without leaving a money trail?

Why is this code 6.5x slower with optimizations enabled?

How can the DM most effectively choose 1 out of an odd number of players to be targeted by an attack or effect?

Can I interfere when another PC is about to be attacked?

What is the command to reset a PC without deleting any files

Why is an old chain unsafe?

Can a German sentence have two subjects?

Email Account under attack (really) - anything I can do?

cryptic clue: mammal sounds like relative consumer (8)

Divisibility of sum of multinomials

How to determine if window is maximised or minimised from bash script

Should I join an office cleaning event for free?

Is there a minimum number of transactions in a block?

Can town administrative "code" overule state laws like those forbidding trespassing?

My colleague's body is amazing

Is there really no realistic way for a skeleton monster to move around without magic?

Do airline pilots ever risk not hearing communication directed to them specifically, from traffic controllers?

I see my dog run

Extreme, but not acceptable situation and I can't start the work tomorrow morning

Is it legal to have the "// (c) 2019 John Smith" header in all files when there are hundreds of contributors?



Infinite sum of harmonic number


Sum of reciprocals of numbers with certain terms omittedWhat is the closed form sum of this series?Is there a name for infinite series of this type?Infinite amount of additions, finite sum?Why can infinite series be summed different ways to get different results?Why does the order of summation of the terms of an infinite series influence its value?Infinite Series $left(frac12+frac14-frac23right)+left(frac15+frac17-frac26right)+left(frac18+frac{1}{10}-frac29right)+cdots$Sum of the recripocals of the Harmonic NumbersFind the sum of the infinite seriesAlternating harmonic series convergence













4












$begingroup$


I learned that I can find the value of some infinite sum.



Then what is the value of this sum?
$$frac12 + left(1+frac12right)frac1{2^2}+left(1+frac12 +frac13right)frac1{2^3}+left(1+frac12 +frac13 +frac14right)frac1{2^4} + cdots $$
And I want to know How to find the value of the infinite sum of this-like form.










share|cite|improve this question









$endgroup$

















    4












    $begingroup$


    I learned that I can find the value of some infinite sum.



    Then what is the value of this sum?
    $$frac12 + left(1+frac12right)frac1{2^2}+left(1+frac12 +frac13right)frac1{2^3}+left(1+frac12 +frac13 +frac14right)frac1{2^4} + cdots $$
    And I want to know How to find the value of the infinite sum of this-like form.










    share|cite|improve this question









    $endgroup$















      4












      4








      4


      3



      $begingroup$


      I learned that I can find the value of some infinite sum.



      Then what is the value of this sum?
      $$frac12 + left(1+frac12right)frac1{2^2}+left(1+frac12 +frac13right)frac1{2^3}+left(1+frac12 +frac13 +frac14right)frac1{2^4} + cdots $$
      And I want to know How to find the value of the infinite sum of this-like form.










      share|cite|improve this question









      $endgroup$




      I learned that I can find the value of some infinite sum.



      Then what is the value of this sum?
      $$frac12 + left(1+frac12right)frac1{2^2}+left(1+frac12 +frac13right)frac1{2^3}+left(1+frac12 +frac13 +frac14right)frac1{2^4} + cdots $$
      And I want to know How to find the value of the infinite sum of this-like form.







      sequences-and-series






      share|cite|improve this question













      share|cite|improve this question











      share|cite|improve this question




      share|cite|improve this question










      asked Apr 2 at 14:47









      S. YooS. Yoo

      424




      424






















          1 Answer
          1






          active

          oldest

          votes


















          8












          $begingroup$

          One can split this summation into
          $$left(frac12+frac1{2^2}+frac1{2^3}+cdotsright)+frac12left(frac1{2^2}+frac1{2^3}+cdotsright)+frac13left(frac1{2^3}+frac1{2^4}+cdotsright)+cdots$$
          $$begin{align}
          &=sum_{k=1}^infty frac1k sum_{j=k}^infty frac1{2^j}\
          &=sum_{k=1}^infty frac1k left(frac{2^{-k}}{1-2^{-1}}right)\
          &=sum_{k=1}^infty frac{2^{1-k}}k\
          &=2sum_{k=1}^infty frac{left(frac12right)^k}k\
          &=2left(-ln{left(1-frac12right)}right)\
          &boxed{=2ln{(2)}}
          end{align}$$

          By using the fact that
          $$ln{(1-x)}=-sum_{k=1}^infty frac{x^k}k$$
          for all $|x|lt 1$.



          In fact one can use a similar method to prove that
          $$sum_{k=1}^infty x^kH_k=frac1{1-x}ln{left(frac1{1-x}right)}$$
          for $|x|lt1$. Where $H_k$ is the $k$th harmonic number given by
          $$H_k=sum_{n=1}^kfrac1n$$






          share|cite|improve this answer











          $endgroup$














            Your Answer





            StackExchange.ifUsing("editor", function () {
            return StackExchange.using("mathjaxEditing", function () {
            StackExchange.MarkdownEditor.creationCallbacks.add(function (editor, postfix) {
            StackExchange.mathjaxEditing.prepareWmdForMathJax(editor, postfix, [["$", "$"], ["\\(","\\)"]]);
            });
            });
            }, "mathjax-editing");

            StackExchange.ready(function() {
            var channelOptions = {
            tags: "".split(" "),
            id: "69"
            };
            initTagRenderer("".split(" "), "".split(" "), channelOptions);

            StackExchange.using("externalEditor", function() {
            // Have to fire editor after snippets, if snippets enabled
            if (StackExchange.settings.snippets.snippetsEnabled) {
            StackExchange.using("snippets", function() {
            createEditor();
            });
            }
            else {
            createEditor();
            }
            });

            function createEditor() {
            StackExchange.prepareEditor({
            heartbeatType: 'answer',
            autoActivateHeartbeat: false,
            convertImagesToLinks: true,
            noModals: true,
            showLowRepImageUploadWarning: true,
            reputationToPostImages: 10,
            bindNavPrevention: true,
            postfix: "",
            imageUploader: {
            brandingHtml: "Powered by u003ca class="icon-imgur-white" href="https://imgur.com/"u003eu003c/au003e",
            contentPolicyHtml: "User contributions licensed under u003ca href="https://creativecommons.org/licenses/by-sa/3.0/"u003ecc by-sa 3.0 with attribution requiredu003c/au003e u003ca href="https://stackoverflow.com/legal/content-policy"u003e(content policy)u003c/au003e",
            allowUrls: true
            },
            noCode: true, onDemand: true,
            discardSelector: ".discard-answer"
            ,immediatelyShowMarkdownHelp:true
            });


            }
            });














            draft saved

            draft discarded


















            StackExchange.ready(
            function () {
            StackExchange.openid.initPostLogin('.new-post-login', 'https%3a%2f%2fmath.stackexchange.com%2fquestions%2f3171942%2finfinite-sum-of-harmonic-number%23new-answer', 'question_page');
            }
            );

            Post as a guest















            Required, but never shown

























            1 Answer
            1






            active

            oldest

            votes








            1 Answer
            1






            active

            oldest

            votes









            active

            oldest

            votes






            active

            oldest

            votes









            8












            $begingroup$

            One can split this summation into
            $$left(frac12+frac1{2^2}+frac1{2^3}+cdotsright)+frac12left(frac1{2^2}+frac1{2^3}+cdotsright)+frac13left(frac1{2^3}+frac1{2^4}+cdotsright)+cdots$$
            $$begin{align}
            &=sum_{k=1}^infty frac1k sum_{j=k}^infty frac1{2^j}\
            &=sum_{k=1}^infty frac1k left(frac{2^{-k}}{1-2^{-1}}right)\
            &=sum_{k=1}^infty frac{2^{1-k}}k\
            &=2sum_{k=1}^infty frac{left(frac12right)^k}k\
            &=2left(-ln{left(1-frac12right)}right)\
            &boxed{=2ln{(2)}}
            end{align}$$

            By using the fact that
            $$ln{(1-x)}=-sum_{k=1}^infty frac{x^k}k$$
            for all $|x|lt 1$.



            In fact one can use a similar method to prove that
            $$sum_{k=1}^infty x^kH_k=frac1{1-x}ln{left(frac1{1-x}right)}$$
            for $|x|lt1$. Where $H_k$ is the $k$th harmonic number given by
            $$H_k=sum_{n=1}^kfrac1n$$






            share|cite|improve this answer











            $endgroup$


















              8












              $begingroup$

              One can split this summation into
              $$left(frac12+frac1{2^2}+frac1{2^3}+cdotsright)+frac12left(frac1{2^2}+frac1{2^3}+cdotsright)+frac13left(frac1{2^3}+frac1{2^4}+cdotsright)+cdots$$
              $$begin{align}
              &=sum_{k=1}^infty frac1k sum_{j=k}^infty frac1{2^j}\
              &=sum_{k=1}^infty frac1k left(frac{2^{-k}}{1-2^{-1}}right)\
              &=sum_{k=1}^infty frac{2^{1-k}}k\
              &=2sum_{k=1}^infty frac{left(frac12right)^k}k\
              &=2left(-ln{left(1-frac12right)}right)\
              &boxed{=2ln{(2)}}
              end{align}$$

              By using the fact that
              $$ln{(1-x)}=-sum_{k=1}^infty frac{x^k}k$$
              for all $|x|lt 1$.



              In fact one can use a similar method to prove that
              $$sum_{k=1}^infty x^kH_k=frac1{1-x}ln{left(frac1{1-x}right)}$$
              for $|x|lt1$. Where $H_k$ is the $k$th harmonic number given by
              $$H_k=sum_{n=1}^kfrac1n$$






              share|cite|improve this answer











              $endgroup$
















                8












                8








                8





                $begingroup$

                One can split this summation into
                $$left(frac12+frac1{2^2}+frac1{2^3}+cdotsright)+frac12left(frac1{2^2}+frac1{2^3}+cdotsright)+frac13left(frac1{2^3}+frac1{2^4}+cdotsright)+cdots$$
                $$begin{align}
                &=sum_{k=1}^infty frac1k sum_{j=k}^infty frac1{2^j}\
                &=sum_{k=1}^infty frac1k left(frac{2^{-k}}{1-2^{-1}}right)\
                &=sum_{k=1}^infty frac{2^{1-k}}k\
                &=2sum_{k=1}^infty frac{left(frac12right)^k}k\
                &=2left(-ln{left(1-frac12right)}right)\
                &boxed{=2ln{(2)}}
                end{align}$$

                By using the fact that
                $$ln{(1-x)}=-sum_{k=1}^infty frac{x^k}k$$
                for all $|x|lt 1$.



                In fact one can use a similar method to prove that
                $$sum_{k=1}^infty x^kH_k=frac1{1-x}ln{left(frac1{1-x}right)}$$
                for $|x|lt1$. Where $H_k$ is the $k$th harmonic number given by
                $$H_k=sum_{n=1}^kfrac1n$$






                share|cite|improve this answer











                $endgroup$



                One can split this summation into
                $$left(frac12+frac1{2^2}+frac1{2^3}+cdotsright)+frac12left(frac1{2^2}+frac1{2^3}+cdotsright)+frac13left(frac1{2^3}+frac1{2^4}+cdotsright)+cdots$$
                $$begin{align}
                &=sum_{k=1}^infty frac1k sum_{j=k}^infty frac1{2^j}\
                &=sum_{k=1}^infty frac1k left(frac{2^{-k}}{1-2^{-1}}right)\
                &=sum_{k=1}^infty frac{2^{1-k}}k\
                &=2sum_{k=1}^infty frac{left(frac12right)^k}k\
                &=2left(-ln{left(1-frac12right)}right)\
                &boxed{=2ln{(2)}}
                end{align}$$

                By using the fact that
                $$ln{(1-x)}=-sum_{k=1}^infty frac{x^k}k$$
                for all $|x|lt 1$.



                In fact one can use a similar method to prove that
                $$sum_{k=1}^infty x^kH_k=frac1{1-x}ln{left(frac1{1-x}right)}$$
                for $|x|lt1$. Where $H_k$ is the $k$th harmonic number given by
                $$H_k=sum_{n=1}^kfrac1n$$







                share|cite|improve this answer














                share|cite|improve this answer



                share|cite|improve this answer








                edited Apr 2 at 18:22









                Théophile

                20.4k13047




                20.4k13047










                answered Apr 2 at 15:08









                Peter ForemanPeter Foreman

                6,2611317




                6,2611317






























                    draft saved

                    draft discarded




















































                    Thanks for contributing an answer to Mathematics Stack Exchange!


                    • Please be sure to answer the question. Provide details and share your research!

                    But avoid



                    • Asking for help, clarification, or responding to other answers.

                    • Making statements based on opinion; back them up with references or personal experience.


                    Use MathJax to format equations. MathJax reference.


                    To learn more, see our tips on writing great answers.




                    draft saved


                    draft discarded














                    StackExchange.ready(
                    function () {
                    StackExchange.openid.initPostLogin('.new-post-login', 'https%3a%2f%2fmath.stackexchange.com%2fquestions%2f3171942%2finfinite-sum-of-harmonic-number%23new-answer', 'question_page');
                    }
                    );

                    Post as a guest















                    Required, but never shown





















































                    Required, but never shown














                    Required, but never shown












                    Required, but never shown







                    Required, but never shown

































                    Required, but never shown














                    Required, but never shown












                    Required, but never shown







                    Required, but never shown







                    Popular posts from this blog

                    is 'sed' thread safeWhat should someone know about using Python scripts in the shell?Nexenta bash script uses...

                    How do i solve the “ No module named 'mlxtend' ” issue on Jupyter?

                    Pilgersdorf Inhaltsverzeichnis Geografie | Geschichte | Bevölkerungsentwicklung | Politik | Kultur...